www.vorhilfe.de
- Förderverein -
Der Förderverein.

Gemeinnütziger Verein zur Finanzierung des Projekts Vorhilfe.de.
Hallo Gast!einloggen | registrieren ]
Startseite · Mitglieder · Impressum
Forenbaum
^ Forenbaum
Status VH e.V.
  Status Vereinsforum

Gezeigt werden alle Foren bis zur Tiefe 2

Navigation
 Startseite...
 Suchen
 Impressum
Das Projekt
Server und Internetanbindung werden durch Spenden finanziert.
Organisiert wird das Projekt von unserem Koordinatorenteam.
Hunderte Mitglieder helfen ehrenamtlich in unseren moderierten Foren.
Anbieter der Seite ist der gemeinnützige Verein "Vorhilfe.de e.V.".
Partnerseiten
Weitere Fächer:

Open Source FunktionenplotterFunkyPlot: Kostenloser und quelloffener Funktionenplotter für Linux und andere Betriebssysteme
Forum "Reelle Analysis mehrerer Veränderlichen" - Fouriertrafo / Faltung
Fouriertrafo / Faltung < mehrere Veränderl. < reell < Analysis < Hochschule < Mathe < Vorhilfe
Ansicht: [ geschachtelt ] | ^ Forum "Reelle Analysis mehrerer Veränderlichen"  | ^^ Alle Foren  | ^ Forenbaum  | Materialien

Fouriertrafo / Faltung: Frage (für Interessierte)
Status: (Frage) für Interessierte Status 
Datum: 00:36 Do 15.12.2005
Autor: martin1984

Eingabefehler: "\left" und "\right" müssen immer paarweise auftreten, es wurde aber ein Teil ohne Entsprechung gefunden (siehe rote Markierung)
Eingabefehler: "\left" und "\right" müssen immer paarweise auftreten, es wurde aber ein Teil ohne Entsprechung gefunden (siehe rote Markierung)

Aufgabe
Ich muss einen Proseminarvortrag am 10.Januar 2006 halten. Thema des Seminares ist die Fouriertransformation. Ich habe folgende Aufgabe:
Beweisen der Formeln für die Operatoren der kinetischen (a) und potentiellen (b) Energie in der Quantenmechanik.

Sei $S(\IR^n)$ der Schwarzraum über $\IR^n$.  Sei $(p,q)\in\IR^{2n}$.
definiere $f_{p,q}(x):=g(x-q)\cdot e^{-ip\cdot x}\quad \Rightarrow\forall (p,q): \left\| f_{p,q}\right\|_2=\left\|g\right\|_2=1. \quad g  \text{bel.}\in S$

Sei $\psi, \varphi\in S(\IR^n) $ u. $\left\langle\psi,\varphi\right\rangle=\int_{\IR^n}\bar{\psi}(x)\varphi(x)dx$

Dann $\left\langle\psi,f_{p,q}\right\rangle=\int\bar{\psi}(x)e^{-ip\cdot x}g(x-q)dx$

Benutze (macht eine andere Gruppe):\\
$\forall \psi,\varphi \in S: \int_{\IR^{2n}}\left\langle\psi,f_{p,q}\right\rangle\left\langle f_{p,q},\varphi\right\rangle\frac{d^n p d^n q}{(2\pi)^n}=\left\langle\psi,\varphi\right\rangle$
(Satz von Plancherel) $\Leftrightarrow :\mathbbm{1}_S=\int_{\IR^{2n}}\frac{d^n p d^n q}{(2\pi)^n}\underbrace{\left|f_{p,q}\right\rangle\left\langle f_{p,q}\right|}_{:=\Pi_{p,q}, \text{Projektion auf} f}$

$(\Pi_{p,q}\varphi=\left\langle f_{p,q},\varphi\right\rangle\cdot f_{p,q}, \qquad \left\langle\psi,\Pi_{p,q}\varphi\right\rangle=\left\langle f_{p,q},\varphi\right\rangle\left\langle\psi,f_{p,q}\right\rangle)$

(a)  Operator der kinetischen Energie in der Quantenmechanik
$\left\langle\psi,-\Delta\varphi\right\rangle = \int_{\IR^{2n}}\left\langle\psi,f_{p,q}\right\rangle\left\langle f_{p,q},\varphi\right\rangle\cdot(p^2+\left\|\nabla g\right\|^2_2)\frac{d^n p d^n q}{(2\pi)^n}$

(b)  Operator der potentiellen Energie in der QM
$\left\langle\psi,(V\*\left|g\right|^2)\varphi\right\rangle = \int_{\IR^{2n}}\left\langle\psi,f_{p,q}\right\rangle\left\langle f_{p,q},\varphi\right\rangle V(q)\frac{d^n p d^n q}{(2\pi)^n}$

$\Delta$ meint hier den Laplace-Operator und $\nabla$ den Nabla-Operator.


Nun zu meinem Lösungsansatz für die erste Formel, der mich aber leider noch nicht zum Erfolg führte:
Mein Prof. meinte, es würde genügen, die Formel 1-dimensional zu zeigen und nur für $\psi=\varphi$ (wegen irgend so einem Parallelogramm-Satz).

$\left\langle \psi, -\Delta\psi\right\rangle=\left\langle \nabla\psi,\nabla\psi\right\rangle$\\
$=\int_{\mathbb{R}^2}\left\langle\nabla\psi, f_{p,q}\right\rangle\left\langle f_{p,q},\nabla\psi\right\rangle \frac{dp dq}{2\pi}$\\
$=\int_{\mathbb{R}^2}\left(\int_{\mathbb{R}}\nabla\bar{\psi}(x) g(x-q)e^{-ipx}dx\int_{\mathbb{R}}\bar{g}(y-q)e^{ipy}\nabla\psi(y)dy\right)\frac{dp dq}{2\pi}$\\

$\underbrace{=}_{\text{part.Integration}}$ $ \int_{\mathbb{R}^2}\left(\int_{\mathbb{R}}\bar{\psi}(x) g'(x-q)e^{-ipx}dx \int_{\mathbb{R}}\psi(y)\bar{g}'(y-q)e^{ipy}dy$\\
$ -ip\int_{\mathbb{R}}\bar{\psi}(x) g(x-q)e^{-ipx}dx \int_{\mathbb{R}}\psi(y)\bar{g}'(y-q)e^{ipy}dy$\\
$+ip\int_{\mathbb{R}}\bar{\psi}(x) g'(x-q)e^{-ipx}dx \int_{\mathbb{R}}\psi(y)\bar{g}'(y-q)e^{ipy}dy$\\
$\underbrace{+p^2\int_{\mathbb{R}}\bar{\psi}(x) g(x-q)e^{-ipx}dx \int_{\mathbb{R}}\psi(y)\bar{g}(y-q)e^{ipy}dy}_{=p^2\left\langle\psi,f_{p,q}\right\rangle\left\langle f_{p,q},\psi\right\rangle}\right)\frac{dp dq}{2\pi}$

Im zweiten Schritt wurde die Formel von oben, die eine andere Gruppe beweis, benutzt.

Nun habe ich den ersten Teil da stehen. Für den zweiten müsste dann gelten, falls ich richtig gerechnet habe:

$ \int_{\mathbb{R}^2}\left\|\nabla g\right\|_2^2\cdot \left\langle\psi,f_{p,q}\right\rangle\left\langle f_{p,q},\varphi\right\rangle\frac{dp dq}{2\pi}$
\\$= \int_{\mathbb{R}^2} \int_{\mathbb{R}}\left|\nabla g(x)\right|^2dx\left\langle\psi,f_{p,q}\right\rangle\left\langle f_{p,q},\varphi\right\rangle\frac{dp dq}{2\pi}$
\\$= \int_{\mathbb{R}^2} \int_{\mathbb{R}}\nabla\bar{g}(x)\nabla g(x)dx \left\langle\psi,f_{p,q}\right\rangle\left\langle f_{p,q},\varphi\right\rangle\frac{dp dq}{2\pi}$\\

$= \int_{\mathbb{R}^2}\int_{\mathbb{R}}\bar{\psi}(x) g'(x-q)e^{-ipx}dx \int_{\mathbb{R}}\psi(y)\bar{g}'(y-q)e^{ipy}dy$\\
$ -ip\int_{\mathbb{R}}\bar{\psi}(x) g(x-q)e^{-ipx}dx \int_{\mathbb{R}}\psi(y)\bar{g}'(y-q)e^{ipy}dy$\\
$+ip\int_{\mathbb{R}}\bar{\psi}(x) g'(x-q)e^{-ipx}dx \int_{\mathbb{R}}\psi(y)\bar{g}(y-q)e^{ipy}dy\frac{dp dq}{2\pi}$

Aber wie kann ich das zeigen? Vielleicht ist hilfreich, dass im letzten Glied der Gleichungskette die letzten bei den Summanden genau komplex kojugiert zueinander auftreten.

Wie gesagt, das Thema des Seminares ist die Fouriertransformation und vielleicht muss man in irgendwo den Satz von Plancherel anwenden (bei (b) sicher), oder diese ganzen Umformungsregeln zwischen Fouriertrafo und Faltung.

Es wäre nett, wenn sich auch zu Formel (b)  jemand äußern könnte!

Vielen Dank!


Ich habe diese Frage in keinem Forum auf anderen Internetseiten gestellt.

        
Bezug
Fouriertrafo / Faltung: Mitteilung
Status: (Mitteilung) Reaktion unnötig Status 
Datum: 20:40 Fr 16.12.2005
Autor: martin1984

Aufgabe
Ich habe vorgestern schon einmal eine Frage gestellt mit dem gleichen Diskussionsthema. In dieser Sache bin ich etwas weitergekommen.
Ich muss nur noch zeigen, dass folgendes gilt:

$ [mm] -ip\int_{\mathbb{R}}\bar{\psi}(x) g(x-q)e^{-ipx}dx \int_{\mathbb{R}}\psi(y)\bar{g}'(y-q)e^{ipy}dy$\\ [/mm]
[mm] $+ip\int_{\mathbb{R}}\bar{\psi}(x) g'(x-q)e^{-ipx}dx \int_{\mathbb{R}}\psi(y)\bar{g}(y-q)e^{ipy}dy$ [/mm]
$=0$

d.h.:

$ [mm] ip\int_{\mathbb{R}}\bar{\psi}(x) g(x-q)e^{-ipx}dx \int_{\mathbb{R}}\psi(y)\bar{g}'(y-q)e^{ipy}dy$\\ [/mm]
$= [mm] ip\int_{\mathbb{R}}\bar{\psi}(x) g'(x-q)e^{-ipx}dx \int_{\mathbb{R}}\psi(y)\bar{g}(y-q)e^{ipy}dy$ [/mm]

Man kann die Therme auch als durch Skalarprodukte der Form  
        [mm] $<\psi,\varphi>=\int \bar{\psi}(x)\varphi(x)dx$ [/mm] ausdrücken.

Kann mir jemand sagen, wie ich das zeige?

Also mir ist aufgefallen, dass die Therme genau komplex konjugiert zueinander sind.

Vielen Dank!

Ich habe diese Frage in keinem Forum auf anderen Internetseiten gestellt.

Bezug
        
Bezug
Fouriertrafo / Faltung: Fälligkeit abgelaufen
Status: (Mitteilung) Reaktion unnötig Status 
Datum: 08:51 Mi 21.12.2005
Autor: matux

Hallo Martin,

[willkommenmr] !!


Leider konnte Dir keiner hier mit Deinem Problem in der von Dir vorgegebenen Zeit weiterhelfen.

Vielleicht hast Du ja beim nächsten Mal mehr Glück [kleeblatt] .


Viele Grüße,
Matux, der Foren-Agent

Allgemeine Tipps wie du dem Überschreiten der Fälligkeitsdauer entgegenwirken kannst findest du in den Regeln für die Benutzung unserer Foren.


Bezug
Ansicht: [ geschachtelt ] | ^ Forum "Reelle Analysis mehrerer Veränderlichen"  | ^^ Alle Foren  | ^ Forenbaum  | Materialien


^ Seitenanfang ^
ev.vorhilfe.de
[ Startseite | Mitglieder | Impressum ]